You are on page 1of 42

Example 1:

i1
FIN 5080 - Jack De Jong: Chapter 7: Stocks, Stock Valuation, and Stock Market Equilibrium

What is the value of a preferred stock where the dividend rate is 16% on a $100 par
value? The required rate of return for a stock of this risk level is 12%.

D ps
Value of Preferred Stock  Vps 
rps
Dividend Amount = $16.00
Required Rate of Return = 12.00%

Value of Preferred Stock = $133.33

Example 2: You own 250 shares of McCormick Resources preferred stock, which currently sells for
$38.50 per share and pays annual dividends of $3.25 per share.
a. What is your expected return?
Dps $3.25
Expected Rate of Return  r̂ps    8.44%
P0 $38.50

b. If you require an 8% return, given the current price, should you sell or buy more?
D ps $3.25
Value of Preferred Stock  Vps    $40.63
rps .08

Example 3: Gilliland Motor, Inc. paid a $3.75 dividend last year. At a constant rate of growth of 6%,
what is the value of the common stock if investors require a 20% return?
D1 D  1  g 
P̂0   0 where g  expected growth rate and rs  g
rs - g rs - g

Most recently paid dividend = $3.7500


Required rate of return = 20.00%
Growth rate = 6.00%
Next expected dividend = $3.9750
Value of Common Stock = $28.3929

What will be the dividend yield and capital gains yield for the next year?
Dividend expected for year 2 = $4.2135
Expected price at end of year 1 = $30.0964
Dividend Yield = Dividend 1/Price 0 = 14.00%
Capital Gains Yield = (Price 1 - Price 0)/Price 0 = 6.00%

Example 4: The common stock of KPD paid $1 in dividends last year. Dividends are expected to
grow at an 8% annual rate for an indefinite number of years.
a. If KPD’s current market price is $25, what is the stock’s expected rate of return?
b. If your required rate of return is 11%, what is the value of the stock for you?
c. Should you make the investment?
D1
Expected Rate of Return  r̂s  g
P0
Most recently paid dividend = $1.0000
Growth rate = 8.00%
Next expected dividend = $1.0800
Current market price = $25.00

Dividend Yield = D1/P0 = 4.32%


Capital Gains Yield = g = 8.00%
Expected Return on KPD = 12.32%

Example 5: Axel Co. had earnings per share of $5.00 five years ago. This year their EPS was
$10.00. They expect their growth to continue as before. What will their growth rate be?

N 5
PV -$5.00
PMT $0.00
FV $10.00
Type 0 I=g 14.87%

Problem 7-18: Reizenstein Trucking (RT) has just developed a solar panel capable of generating 200%
more electricity than any solar panel currently on the market. As a result, RT is
expected to experience a 15% annual growth rate for the next 5 years. By the end of
5 years, other firms will have developed comparable technology, and RT's growth rate
will slow to 5% per year indefinitely. Stockholders require a return of 12% on RT's
stock. The most recent annual dividend, D0, which was paid yesterday, was $1.75
per share. What's the value of the stock today?

Year Growth Rate Dividend Cash Flows


0 $1.7500
1 15.00% $2.0125 $2.0125
2 15.00% $2.3144 $2.3144
3 15.00% $2.6615 $2.6615
4 15.00% $3.0608 $3.0608
5 15.00% $3.5199 $56.3180
6 5.00% $3.6959

Required Rate of Return on RT = 12.00%


Price at End of Year 5 = D6/(rs - g) = $52.7981

Find Price Today using NPV: I 12.00%


NPV $39.44
D1
Volatility Example: P̂0 
rs - g
Next expected dividend = $2.00
Growth Rate = 5.00%
Required rate of return = 10.00%

Value of Common Stock = $40.00

Required Rate Growth Rates


of Return 4.00% 5.00% 6.00%
9.00% $40.00 $50.00 $66.67
10.00% $33.33 $40.00 $50.00
11.00% $28.57 $33.33 $40.00
n a $100 par

urrently sells for

or buy more?

of growth of 6%,

re expected to

te of return?
r growth rate be?

generating 200%

By the end of
RT's growth rate
FIN 5080 - Jack De Jong: Chapter 7: Stocks, Stock Valuation, and Stock Market Equilibrium

MB8:
Problem 7-5:
D1 D2 DN PN D
P̂0     where PN  N 1
1  rs  1
1  rs  2
1  rs  N
1  rs  N
rs - g

CAPM: ri  rRF   rM - rRF   b i  rRF   RPM   b i

Year Growth Rate Dividend Cash Flows


0 $2.0000
1 20.00% $2.4000 $2.4000
2 20.00% $2.8800 $61.0234
3 7.00% $3.0816

Risk-free rate 7.50%


Market risk prem. 4.00%
Beta 1.2000
RRR on Stock 12.30%

P2 = D3/(rs - g) $ 58.1434
P0 hat = NPV $ 50.5250

Year Growth Rate Dividend Cash Flows


0 $ 2.0000
1 20.00% $ 2.4000 $ 2.4000
2 20.00% $ 2.8800 $ 2.8800
3 7.00% $ 3.0816 $ 65.2950
4 7.00% $ 3.2973
Risk-free rate 7.50%
Market risk prem. 4.00%
Beta 1.2000
RRR on Stock 12.30%

P3 = D4/(rs - g) $ 62.2134
P0 = NPV $ 50.5250

Problem 7-15:
D1 D  1  g 
P̂0   0 where g  expected growth rate and rs  g
rs - g rs - g

Part a:
RRR = 15.00%
D0 = $ 2.00

Growth Rates P0 Hat


-5.00% $ 9.50 Part a
0.00% $ 13.33
5.00% $ 21.00
10.00% $ 44.00
15.00% #DIV/0! Part b
20.00% $ (48.00)

Part c: NO, because g > RRR is not sustainable.

Problem 7-12:
D1 D2 DN PN D
P̂0     where PN  N 1
1  rs  1  rs 
1 2
1  rs  N
1  rs  N
rs - g

Year Growth Rate Dividend Cash Flows


0 $ -
1 $ - $ -
2 $ - $ -
3 $ 1.0000 $ 1.0000
4 50.00% $ 1.5000 $ 1.5000
5 50.00% $ 2.2500 $ 36.9643
6 8.00% $ 2.4300

RRR = 15.00%
P5 = D6/(rs - g) $ 34.71

P0 = NPV $ 19.89 $ 26.31 Wrong answer if do not include the $0


dividends in years 1 and 2.
Problem 7-11:
D1
Expected Rate of Return  r̂s  g
P0
RRR = 13.00%

Year Growth Rate Dividend Cash Flows


0 $1.0000
1 50.00% $1.5000 $1.5000
2 25.00% $1.8750 $30.2679
3 6.00% $1.9875

P2 = D3/(rs - g) = $ 28.3929

P0 Hat = NPV $ 25.0316


FIN 5080-Jack De Jong: Chapter 7: Stocks, Stock Valuation, and Stock Market Equilibrium
Quiz #11: May 12, 2011

1. C Molen Inc. has an outstanding issue of perpetual preferred stock with an annual dividend of
$7.50 per share. It is selling in the marketplace for $117.25 and your required rate of return
on this preferred stock is 6.5%. Which of the following statements is true?
D ps
Value of Preferred Stock  Vps 
rps
D ps
Expected Rate of Return  r̂ps 
P0
Annual dividend of preferred = $ 7.50
Required rate of return on P/S = 6.50%
Current market price of P/S = $ 117.25

Intrinsic Value of P/S = $ 115.38


Expected rate of return on P/S = 6.40%

Molen's P/S is overvalued, so you would sell it if you owned it.

2. B Schnusenberg Corporation just paid a dividend of $0.75 per share, and that
dividend is expected to grow at a constant rate of 6.50% per year in the future. The
company’s beta is 1.25, the required return on the market is 10.50%, and the
risk–free rate is 4.50%. What is the intrinsic value of the company’s stock?

CAPM: ri  rRF   rM - rRF   b i  rRF   RPM   b i


D1 D  1  g 
P̂0   0 where g  expected growth rate and rs  g
rs - g rs - g
D0 = $ 0.7500
g= 6.50%
D1 = $ 0.7988
Risk-free rate = 4.50%
Market required rate = 10.50%
Schnusenberg's beta = 1.25
Schnusenberg's RRR = 12.00%

Schnusenberg's Intrinsic Value = P0 hat = $ 14.52


FIN 5080-Jack De Jong: Chapter 7: Stocks, Stock Valuation, and Stock Market Equilibrium
Quiz #12: May 17, 2011

1. E A stock just paid a dividend of $1.75 per share, has a constant growth rate of 3.6%, and is
currently selling for $32.00 per share. What is the stock’s expected total return for the
upcoming year?
D1
Expected Rate of Return  r̂s  g
P0

D0 = $ 1.7500
g= 3.60%
D1 = $ 1.8130
P0 = $ 32.0000

Dividend Yield = 5.67%


Capital Gains Yield = 3.60%
Expected rate of return = 9.27%

2. A Ackert Company’s last dividend was $1.55. The dividend growth rate is
expected to be constant at 1.5% for the next two years, after which the dividends are
expected to grow at a rate of 8% forever. The firm’s required rate of return is 12%.
What is the intrinsic value of the company’s stock?
D1 D2 DN PN D
P̂0     where PN  N 1
1  rs  1  rs 
1 2
1  rs  N
1  rs  N
rs - g

Year Growth Rate Dividend Cash Flows


0 $ 1.5500
1 1.50% $ 1.5733 $ 1.5733
2 1.50% $ 1.5968 $ 44.7118
3 8.00% $ 1.7246

RRR on Ackert = 12.00%


P2 = D3/(rs - g) = $ 43.1149

Find Price Today = NPV: I 12.00%


NPV $ 37.0486

Other Calculations:
P3=P2*(1+g) $ 46.5641
P1=NPV $ 39.9212

Year 1: Dividend Yield = 4.25%


Capital Gains Yield = (P1 - P0)/P0 = 7.75%
Total Expected Return in Year 1 = 12.00%
Example 1:
i1
FIN 5080 - Jack De Jong: Chapter 9: The Cost of Capital

If we were to get our capital through a bank note where the interest rate is 11% and our
marginal tax rate is 34%, what would be the cost of our loan?

After - tax Cost of Debt  rd  1 - T 

Before-tax Cost of Note =- 11.00%


Marginal Tax Rate = 40.00%

After-tax Cost of Debt = 6.60%

Problem 9-9: A company’s 6% coupon rate, semiannual payment, $1,000 par value bond that matures
in 30 years sells at a price of $515.16. The company’s federal–plus–state tax rate is
40%. What is the firm’s component cost of debt for purposes of calculating the WACC?

VB  1 - F  
2N INT 2  
M
t 2N
t 1  rd   rd 
1  2  1  2 
   
Find YTM and then adjust for taxes:
N 60
PV ($515.16)
PMT $30.00
FV $1,000.00
Type 0 I = YTM 6.00%

6 Month YTM = 6.00%


Nominal Annual YTM = 12.00%
Marginal Tax Rate = 40.00%

After-tax Cost of Debt = 7.20%

What if 2% flotation costs were expected on issuing new bonds?


Marginal Tax Rate = 40.00%
Flotation Costs as a % = 2.00%
N 60
PV ($504.86)
PMT $30.00
FV $1,000.00
Type 0 I 6.12%

6 Month Before-tax Cost of Debt = 6.12%


Annual Before-tax Cost of Debt = 12.23%
After-tax Cost of Debt = 7.34%

What happens if the tax rate decreases to 25%?


Marginal Tax Rate = 25.00%
Flotation Costs as a % = 2.00%

After - tax Cost of Debt  rd  1 - T 


After-tax Cost of Debt = 12.23%*(1-.25)= 9.18%

Problem 9-17(2): TII’s perpetual preferred stock has a $100 par value, pays a quarterly dividend of $2, and
has a yield to investors of 11%. New perpetual preferred stock would have to provide the
same yield to investors, and the company would incur a 5% flotation cost to sell it. What
is the component cost of preferred stock for purposes of calculating the WACC?
D ps D ps
Expected Rate of Return  r̂ps   Pps 
Pps r̂ps

Quarterly Preferred Dividend = $2.00


Annual Preferred Dividend = $8.00
Yield on Preferred Stock = 11.00%

Market Price of Preferred Stock = $72.73


D ps
rps 
Pps  1 - F

Percentage Flotation Costs = 5.00%

Comp. Cost of Preferred Stock = 11.58%

Problem 9-17(5): Betas, as reported by security analysts, range from 1.3 to 1.7; the T–bond rate is 10%;
and the market risk premium is estimated by various brokerage houses to be in the range
of 4.5% to 5.5%. What is the component cost of retained earnings?

rs  rRF   rM - rRF   b i  rRF   RPM   b i


Lowest Midpoint Highest
Risk-free Rate 10.00% 10.00% 10.00%
Market Risk Prem. 4.50% 5.00% 5.50%
TII's Beta 1.30 1.50 1.70

Cost of Equity = 15.85% 17.50% 19.35%

Problem 9-17(4): The company has 4 million shares of common stock outstanding. The stock’s price =
$20, but the stock has recently traded in the range of $17 to $23. The most recently paid
dividend was $1.00 and the corresponding EPS was $2.00. ROE based on average
equity was 24% in 2006, but management expects to increase this return on equity to
30%; however, security analysts are not aware of management’s optimism in this regard.
What is the component cost of retained earnings?

g  ROE   Retention Rate 


Net Income Available to Common Stockholders
where: ROE 
Total Common Equity

Retention Rate  1 - Dividend Payout Ratio


D0  $1.00

EPS 0  $2.00

Dividend Payout Ratio = 50.00%


Retention Rate = 50.00%
ROE = 24.00%

Growth Rate g = 12.00%


D1
rs   Expected g where D1  D 0  1  g 
P0

D 1  D 0  1  g   $1.1200

P0  $20.00

Dividend Yield = 5.60%


Expected Growth Rate = 12.00%

Cost of Equity = 17.60%


Lowest Midpoint Highest
Brokers' Forecasts of g = 10.00% 12.50% 15.00%
Next expected dividend = $1.10 $1.13 $1.15
Dividend Yield = 5.50% 5.63% 5.75%

Cost of Equity = 15.50% 18.13% 20.75%

Problem 9-17(5): Assume flotation costs for issuing new common stock are 5%. What is the component
cost for new common equity?
D1
re   Expected g where : D1  D 0  1  g  and F = percentage flotation costs.
P0  1 - F 

D 1  D 0  1  g   $1.1200

P00  1 - F   $19.00

F = percentage flotation costs = 5.00%


Dividend Yield = 5.89%
Expected Growth Rate = 12.00%

Cost of Equity = 17.89%


Lowest Midpoint Highest
Brokers' Forecasts of g = 10.00% 12.50% 15.00%
Next expected dividend = $1.10 $1.13 $1.15
Dividend Yield = 5.79% 5.92% 6.05%

Cost of Equity = 15.79% 18.42% 21.05%

Problem 9-17(6): At a recent conference, TII’s financial VP polled some pension fund investment managers
on the minimum rate of return they would have to expect on TII’s common to make them
willing to buy the common rather than TII’s bonds, when the bonds yielded 12%. The
responses suggested a risk premium over TII’s bonds of 4% to 6%. What is the
component cost of retained earnings?

rs  Bond Yield  Bond Risk Premium


Lowest Midpoint Highest
TII's Bond Yield = 12.00% 12.00% 12.00%
Bond Risk Premium = 4.00% 5.00% 6.00%

Cost of Equity = 16.00% 17.00% 18.00%

Example 2: MEC uses only debt and equity. It can borrow at 10% if it continues to use 45% debt
and 55% common equity. Its last dividend was $2, its expected growth rate is 4%, and
the stock sells at $20. MEC's marginal tax rate is 40%. What is its WACC?

Before-tax cost of debt = 10.00%


Marginal tax rate = 40.00%
After-tax cost of debt = 6.00%

Last dividend = $2.00


Expected growth rate = 4.00%
Next expected dividend = $2.08
Current stock price = $20.00
Dividend yield = 10.40%
Capital gains yield = 4.00%
Cost of retained earnings = 14.40%

Target Weights Component Costs Wt x Comp. Cost


Debt 45.00% 6.00% 2.70%
Common Stock 55.00% 14.40% 7.92%
WACC 100.00% 10.62%

Problem 9-17: Compute TII's WACC.


Travellers Inn: December 31, 2009 (Millions of Dollars)
Cash $ 10 Accts. Payable $ 10
Accounts Rec. $ 20 Accruals $ 10
Inventories $ 20 ST Debt $ 5
Current Assets $ 50 Current Liabilities $ 25
Net Fixed Assets $ 50 LT Debt $ 30
Total Assets $ 100 Preferred Stock $ 5
Common Stock $ 10
Retained Earn. $ 30
Total Comm. Eq. $ 40
Total Liab. & Eq. $ 100

LT Debt:
N 40
I 6.00%
PMT $40.00
FV $1,000.00
Type 0 PV -$699.07

Market Value of LT Debt = $20,972,221.88

Preferred Stock:
Market Price of Preferred Stock = $72.73
Number of Shares of Preferred = 50,000

Market Value of Preferred Stock = $3,636,363.64

Common Stock:
Market Price of Common Stock = $20.00
Number of Shares of Common = 4,000,000

Market Value of Common Stock = $80,000,000.00

Market Values Weights Component Costs Wt.x Comp.Cost


LT Debt $ 20,972,221.88 20.05% 7.20% 1.44%
Preferred Stock $ 3,636,363.64 3.48% 11.58% 0.40%
Common Stock $80,000,000.00 76.48% 17.60% 13.46%
Total $ 104,608,585.51 100.00% WACC = 15.31%
his regard.
nt managers
FIN 5080 - Jack De Jong: Chapter 9: The Cost of Capital

MB8:
Problem 9-15:
Part a: 50.00% equity
Part b:
Before-tax cost of debt = kd = 8.00%
Marginal tax rate = 40.00%
After-tax cost of debt = 4.80%
Cost of retained earnings = ks = 12.00%

Weights Component Costs Wt X Comp. Cost


Debt 50.00% 4.80% 2.40%
Retained Earnings 50.00% 12.00% 6.00%
WACC 8.40%
Sumproduct 8.40%
Part c: Component cost of new C/S is higher than R/E due to FC and so
WACC will increase.

MC o(2):
N 30
PV $ (980.00)
PMT $ 100.00
FV $ 1,000.00
Type 0 Rate = kd = 10.22%

After tax cost of debt = 6.13%

MC part l:
After tax cost of debt = 7.20%
Risk-free rate 5.60%
Market risk premium 6.00%
Divisional beta 1.70
Cost of R/E for division 15.80%

Weights Component Costs Wt X Comp. Cost


Debt 10.00% 7.20% 0.72%
Retained Earnings 90.00% 15.80% 14.22%
WACC 14.94%
Sumproduct 14.94%
FIN 5080-Jack De Jong: Chapter 9: The Cost of Capital
Quiz #13: May 19, 2011

1. D A company’s perpetual preferred stock currently sells for $92.50 per share, and it pays an
$8.00 annual dividend. If the company were to sell a new preferred issue, it would incur a
flotation cost of 5.00% of the issue price. What is the firm’s cost of preferred stock?
D ps
rps 
Pps  1 - F 

Dividend on P/S = $ 8.0000


Market Price of P/S = $ 92.5000
Flotation Cost = 5.00%
Net Proceeds of P/S = $ 87.8750

Cost of P/S = 9.10%

Investors Expected Return 8.65%

2. D Several years ago the Jakob Company sold a $1,000 par value, noncallable bond that now
has 20 years to maturity and a 7.00% annual coupon that is paid semiannually. The bond
currently sells for $925 and the company’s tax rate is 40%. If the company were to issue
new bonds, they would pay flotation costs of 2% of the bond issue price. What is the
component cost of debt for use in the WACC calculation?

VB  1 - F  
2N INT 2  
M
t 2N After - tax Cost of Debt  rd  1 - T 
t 1  rd   rd 
1  2  1  2 
   

NPER 40
PV $ (906.50)
PMT $ 35.00
FV $ 1,000.00
Type 0 RATE = 6 Mo.kd 3.9703%
Annual kd = 7.9406%

After - tax Cost of Debt  rd  1 - T  4.7644%


FIN 5080-Jack De Jong: Chapter 9: The Cost of Capital
Quiz #14: May 24, 2011

1. B Weaver Chocolate Co. expects to earn $3.50 per share during the upcoming year, its
dividend payout ratio is 65%, its expected constant dividend growth rate is 6.0%, and its
common stock currently sells for $32.50 per share. New stock can be sold to the public
at the current price, but a flotation cost of 5% will be incurred. What will be the cost of
equity from new common stock?
D1
re   Expected g where : D1  D 0  1  g 
P0  1 - F 

EPS1 = $ 3.50
Dividend Payout Ratio = d 65.00%
D1 = $ 2.28
P0 = $ 32.50
F= 5.00%
P0 x (1 - F)= $ 30.88
Growth g = 6.00%
Cost of New C/S = re = 13.37%

Cost of R/E = rs = 13.00%

2. C You were hired as a consultant to Quigley Company, whose target capital structure is 35%
debt, 10% preferred, and 55% common equity. The interest rate on new debt is 6.50%, the
yield on preferred is 6.00%, the cost of common from retained earnings is 11.25%, and the
marginal tax rate is 40%. The firm will not be issuing any new common stock. What is
Quigley’s WACC?

WACC  w d  rd  1 - T   w ps  rps  w ce  rs

Target Wt. Component Cost Wt. x Comp.Cost


Debt 35.00% 3.90% 1.37%
P/S 10.00% 6.00% 0.60%
R/E 55.00% 11.25% 6.19%
Total 100.00% WACC = 8.15%
Sumproduct = 8.15%
FIN 5080 - Jack De Jong: Chapter 10: Basics of Capital Budgeting: Evaluating Cash Flows

Problem 10-21: Your division is considering two investment projects, each of which requires an upfront
expenditure of $25 million. You estimate the cost of capital is 10% and that the
investments will produce the following after-tax cash flows (in millions of dollars):

Year Project A Project B Project C


0 $ (25) $ (25) $ (25)
1 $ 5 $ 20 $ 12
2 $ 10 $ 10 $ 12
3 $ 15 $ 8 $ 12
4 $ 20 $ 6 $ 12
A's Cumulative
Payback: Year Project A Cash Inflows Project B
0 $ (25) $ (25)
1 $ 5 $ 5 $ 20
2 $ 10 $ 15 $ 10
3 $ 15 $ 30 $ 8
4 $ 20 $ 50 $ 6

Project A's Payback = 2 + ((25 - 15)/15) = 2.6667

Project B's Payback = 1 + ((25 - 20)/10) = 1.5000

Project C's Payback = Initial Cost / Annual Cash Inflow = 2.0833

Discounted Payback: Cost of capital = 10.00%


PV of A's Cum. PV of
Year Project A Cash Inflows Cash Inflows
0 $ (25) $ (25,000,000)
1 $ 5 $ 4,545,455 $ 4,545,455
2 $ 10 $ 8,264,463 $ 12,809,917
3 $ 15 $ 11,269,722 $ 24,079,639
4 $ 20 $ 13,660,269 $ 37,739,908
Using Excel's PERCENTRANK fcn:
Project A's Discounted Payback = 3 + ((25M - 24.08M)/13.66M) = 3.0674

PV of B's Cum. PV of
Year Project B Cash Inflows Cash Inflows
0 $ (25) $ (25,000,000)
1 $ 20 $ 18,181,818 $ 18,181,818
2 $ 10 $ 8,264,463 $ 26,446,281
3 $ 8 $ 6,010,518 $ 32,456,799
4 $ 6 $ 4,098,081 $ 36,554,880
Using Excel's PERCENTRANK fcn:
Project B's Discounted Payback = 1 + ((25M - 18.18M)/8.26M) = 1.8250

Project C's Discounted Payback


I 10.00%
PV ($25.00)
PMT $12.00
FV $0.00
Type 0 N = Disc. PB = 2.4511 years

Net Present Value: Cost of capital = 10.00%


Year Project A Project B Project C
0 $ (25) $ (25) $ (25)
1 $ 5 $ 20 $ 12
2 $ 10 $ 10 $ 12
3 $ 15 $ 8 $ 12
4 $ 20 $ 6 $ 12
NPV: $ 12.7399 $ 11.5549 $ 13.0384
$ 12.7399
Profitability Index: Cost of capital = 10.00%

Year Project A Project B Project C


0 $ (25) $ (25) $ (25)
1 $ 5 $ 20 $ 12
2 $ 10 $ 10 $ 12
3 $ 15 $ 8 $ 12
4 $ 20 $ 6 $ 12
NPV: $ 12.7399 $ 11.5549 $ 13.0384
PI: 1.5096 1.4622 1.5215
1.5096 1.4622 1.5215
Internal Rate of Return: Cost of capital = 10.00%

Year Project A Project B Project C


0 $ (25) $ (25) $ (25)
1 $ 5 $ 20 $ 12
2 $ 10 $ 10 $ 12
3 $ 15 $ 8 $ 12
4 $ 20 $ 6 $ 12
NPV: $ 12.7399 $ 11.5549 $ 13.0384
PI: 1.5096 1.4622 1.5215
IRR: 27.27% 36.15% 32.36%
36.15%
Modified IRR: Cost of capital = 10.00%

Year Project A Project B Project C


0 $ (25) $ (25) $ (25)
1 $ 5 $ 20 $ 12
2 $ 10 $ 10 $ 12
3 $ 15 $ 8 $ 12
4 $ 20 $ 6 $ 12
NPV: $ 12.7399 $ 11.5549 $ 13.0384
PI: 1.5096 1.4622 1.5215
IRR: 27.27% 36.15% 32.36%
MIRR: 21.93% 20.96% 22.17%
20.96% 22.17%
36.15% 32.36%

NPV Profile:
Difference in
Year Project A Project B CFs: A - B
0 $ (25) $ (25) $0
1 $ 5 $ 20 ($15)
2 $ 10 $ 10 $0
3 $ 15 $ 8 $7
4 $ 20 $ 6 $14
Crossover Rate = IRR for Differences in CFs = 13.53%

Cost of Capital Project A Project B


0.00% $25.0000 $19.0000
5.00% $18.2438 $14.9648
10.00% $12.7399 $11.5549
13.53% $9.4564 $9.4564
15.00% $8.2071 $8.6434
20.00% $4.4367 $6.1343
25.00% $1.2720 $3.9536
30.00% ($1.4066) $2.0439
35.00% ($3.6913) $0.3597
40.00% ($5.6539) ($1.1349)

NPV Profile

$30.0000
$25.0000
$20.0000
NPV

$15.0000 Proje
$10.0000 Proje
$5.0000
$0.0000
($5.0000) % % 10 13 15 20 25 30 35 40
00 00
0.
($10.0000) 5.
Cost of Capital

Unequal Lives Example: Suppose a firm is considering two mutually exclusive projects with a
cost of capital of 15%. Equipment S has an initial investment of
$20,000 and cash inflows of $12,590 for 3 years. Equipment L has an
initial investment of $20,000 and cash inflows of $6,625 for 9 years.
Which project should you choose, if any?

Unequal Lives Example: Cost of capital = 15.00%

Year Equipment S Equipment L


0 $ (20,000) $ (20,000)
1 $ 12,590 $ 6,625
2 $ 12,590 $ 6,625
3 $ 12,590 $ 6,625
4 $ 6,625
5 $ 6,625
6 $ 6,625
7 $ 6,625
8 $ 6,625
9 $ 6,625
NPV $8,745.80 $11,611.74
IRR 40.02% 30.00%

Unequal Lives Example: Cost of capital = 15.00%

Replacement Chain
Year Equipment S Equipment L
0 $ (20,000) $ (20,000)
1 $ 12,590 $ 6,625
2 $ 12,590 $ 6,625
3 $ (7,410) $ 6,625
4 $ 12,590 $ 6,625
5 $ 12,590 $ 6,625
6 $ (7,410) $ 6,625
7 $ 12,590 $ 6,625
8 $ 12,590 $ 6,625
9 $ 12,590 $ 6,625
NPV $18,277.36 $11,611.74
IRR 40.02% 30.00%

Unequal Lives Example: Cost of capital = 15.00%

Year Equipment S Equipment L


0 $ (20,000) $ (20,000)
1 $ 12,590 $ 6,625
2 $ 12,590 $ 6,625
3 $ 12,590 $ 6,625
4 $ 6,625
5 $ 6,625
6 $ 6,625
7 $ 6,625
8 $ 6,625
9 $ 6,625
NPV $8,745.80 $11,611.74
IRR 40.02% 30.00%

Equivalent Annual Annuity or EAA:


Equipment S:
N 3
I 15.00%
PV = -NPV ($8,745.80)
FV $0.00
Type 0 PMT = EAA $3,830.46

Equipment L:
N 9
I 15.00%
PV = -NPV ($11,611.74)
FV $0.00
Type 0 PMT = EAA $2,433.52
B's Cumulative
Cash Inflows Alt. PB for B
$ (25)
$ 20 $ (5)
$ 30 $ 5
$ 38 $ 13
$ 44 $ 19

years

years 1.5000 years


1.5000
years

Cum. PV of CFs
$ (25,000,000)
$ (20,454,545)
$ (12,190,083)
$ (920,361)
$ 12,739,908
3.0674 years
years

Cum. PV of CFs
$ (25,000,000)
$ (6,818,182)
$ 1,446,281
$ 7,456,799
$ 11,554,880
1.82500 years
years
Project A
Project B

35 40

projects with a

ipment L has an
5 for 9 years.
FIN 5080 - Jack De Jong: Chapter 10: Basics of Capital Budgeting: Evaluating Cash Flows

MB8:
Problem 10-14:
Year Plan A Plan B Plan B - Plan A
0 $ (10.00) $ (10.00) $ -
1 $ 12.00 $ 1.75 $ (10.25)
2 $ - $ 1.75 $ 1.75
3 $ - $ 1.75 $ 1.75
4 $ - $ 1.75 $ 1.75
5 $ - $ 1.75 $ 1.75
6 $ - $ 1.75 $ 1.75
7 $ - $ 1.75 $ 1.75
8 $ - $ 1.75 $ 1.75
9 $ - $ 1.75 $ 1.75
10 $ - $ 1.75 $ 1.75
11 $ - $ 1.75 $ 1.75
12 $ - $ 1.75 $ 1.75
13 $ - $ 1.75 $ 1.75
14 $ - $ 1.75 $ 1.75
15 $ - $ 1.75 $ 1.75
16 $ - $ 1.75 $ 1.75
17 $ - $ 1.75 $ 1.75
18 $ - $ 1.75 $ 1.75
19 $ - $ 1.75 $ 1.75
20 $ - $ 1.75 $ 1.75
Crossover Rate = IRR of Diff. in CFs 16.07%
NPV $ 0.34 $ 0.34
Part c: Yes, WACC is opportunity cost and appropropriate discount rate.
Part d: NPV Profile:
WACC NPV A NPV B
0.00% $ 2.00 $ 25.00
10.00% $ 0.91 $ 4.90
16.07% $ 0.34 $ 0.34
20.00% $ - $ (1.48)
30.00% $ (0.77) $ (4.20)
40.00% $ (1.43) $ (5.63)

Problem 10-4:
Year Project's CFs Cum. CFs Cum CFs
0 $ (52,125.00) $ (52,125.00)
1 $ 12,000.00 $ 12,000.00 $ (40,125.00)
2 $ 12,000.00 $ 24,000.00 $ (28,125.00)
3 $ 12,000.00 $ 36,000.00 $ (16,125.00)
4 $ 12,000.00 $ 48,000.00 $ (4,125.00)
5 $ 12,000.00 $ 60,000.00 $ 7,875.00
6 $ 12,000.00 $ 72,000.00 $ 19,875.00
7 $ 12,000.00 $ 84,000.00 $ 31,875.00
8 $ 12,000.00 $ 96,000.00 $ 43,875.00
WACC = 12.00% Percentrank = PB 4.3438
NPV = $ 7,486.68
PI = 1.1436 1.1436
PB = IO/PMT 4.3438 years
Unrecovered Cost at Start of Year
Payback  Number of Years Prior to Full Recovery 
Cash Flow during Full Recovery Year
4.3438 years

Disc. PB
I = WACC 12.00%
PV = IO $ (52,125.00)
PMT $ 12,000.00
FV $ -
Type 0 NPER = Disc. PB 6.4995 years
WACC = 12.00%
Year Project's CFs PV of CFs Cum PV of CFs
0 $ (52,125.00) $ (52,125.00)
1 $ 12,000.00 $10,714.29 $ (41,410.71)
2 $ 13,000.00 $10,363.52 $ (31,047.19)
3 $ 14,000.00 $9,964.92 $ (21,082.27)
4 $ 15,000.00 $9,532.77 $ (11,549.50)
5 $ 16,000.00 $9,078.83 $ (2,470.67)
6 $ 17,000.00 $8,612.73 $ 6,142.06
7 $ 18,000.00 $8,142.29 $ 14,284.35
8 $ 19,000.00 $7,673.78 $ 21,958.13
Discounted PB = Percentrank x 8 5.2869 years
years

at Start of Year
ull Recovery Year
FIN 5080-Jack De Jong: Chapter 10: Basics of Capital Budgeting: Evaluating Cash Flows
Quiz #15: May 26, 2011

1. B Fernando Designs (FD) is considering a project that has an initial outlay of $900 at Year 0 with
cash inflows of $500 annually at the ends of Year 1, Year 2, and Year 3. If FD’s WACC is
10.00%, what is the project’s discounted payback?

Year Cash Flow


0 $ (900.00)
1 $ 500.00
2 $ 500.00
3 $ 500.00

I = WACC 10.00%
PV = IO $ (900.00)
PMT $ 500.00
FV $ -
Type 0 NPER = Disc. PB 2.0822 years

Ordinary PB = IO / PMT = 1.8000 years

2. D Ehrmann Data Systems (EDS) is considering a project that has an initial outlay of $1,000 at
Year 0 with cash inflows of $450 annually at the ends of Year 1, Year 2, and Year 3. If EDS’s
WACC is 10%, what is the project’s MIRR?

Year Cash Flow


0 $ (1,000.00)
1 $ 450.00
2 $ 450.00
3 $ 450.00
WACC = 10.00%
MIRR = 14.20%
IRR = 16.65%
NPV = $ 119.08
PI = 1.1191

3. C A firm is considering Projects S and L which are mutually exclusive, equally risky, and not
repeatable. The CEO wants to use the IRR criterion, while the CFO favors the NPV method.
The firm’s WACC is 6%. Project S has an initial cash outlay at Year 0 of $1,025 with cash
inflows of $380 annually at the end of Years 1, 2, 3, and 4. Project L has an initial cash outlay
at Year 0 of $2,150 with cash inflows of $765 annually at the end of Years 1, 2, 3, and 4. If
the wrong decision criterion is used, how much potential value would the firm lose?
Project S Project L Difference in
Year Cash Flows Cash Flows CFs: L - S
0 $ (1,025.00) $ (2,150.00) $ (1,125.00)
1 $ 380.00 $ 765.00 $ 385.00
2 $ 380.00 $ 765.00 $ 385.00
3 $ 380.00 $ 765.00 $ 385.00
4 $ 380.00 $ 765.00 $ 385.00
WACC = 6.00%
IRR = 17.86% 15.78% 13.86% Crossover Rate
NPV = $ 291.74 $ 500.81
MIRR = 12.85% 11.70%
Value Lost = NPV L - NPV S = $ 209.07
  
FIN 5080 - Jack De Jong: Chapter 11: Cash Flow Estimation and Risk Analysis

Major Classes and Asset Lives for MACRS:


NPV Profile

Class Type of Property


3 year Certain special manufacturing tools
5 year Automobiles, light duty trucks, computers, and certain special
manufacturing equipment
7 year Most industrial equipment, office furniture, and fixtures
10 year Certain longer-lived types of equipment
27.5 year Residential rental real property like apartment buildings
39 year All nonresidential real property, including commercial and
industrial buildings

Recovery Allowance Percentage for Personal Property:


Ownership Class of Investment
Year 3 Year 5 Year 7 Year 10 Year
1 33% 20% 14% 10%
2 45% 32% 25% 18%
3 15% 19% 17% 14%
4 7% 12% 13% 12%
5 11% 9% 9%
6 6% 9% 7%
7 9% 7%
8 4% 7%
9 7%
10 6%
11 3%
100% 100% 100% 100%

Example #1: The Chung Chemical Corporation is considering the purchase of a chemical analysis
machine. Although the machine being considered will result in an increase in EBIT of
$35,000 per year, it has a purchase price of $100,000, and it will cost an additional
$5,000 after tax to properly install this machine. In addition to properly operate this
machine, inventory must be increased by $5,000. This machine has an expected life of
5 years, after which it will be sold for $10,000. It will be depreciated by MACRS as 5
year property. Assume a 34% marginal tax rate and a WACC of 15%.
a. What is the initial outlay associated with this project?
b. What are the annual operating cash flows associated with this project for years 1
through 5?
c. What are the additional (nonoperating) cash flows in year 5?
d. Should this machine be purchased?
Part a:
Cost of new machine $ 100,000.00
Installation and shipping $ 5,000.00
Increase to NOWC $ 5,000.00
Initial Outlay $ 110,000.00

Part b:
Year MACRS % Depreciation Book Value
0 $ 105,000.00
1 20% $ 21,000.00 $ 84,000.00
2 32% $ 33,600.00 $ 50,400.00
3 19% $ 19,950.00 $ 30,450.00
4 12% $ 12,600.00 $ 17,850.00
5 11% $ 11,550.00 $ 6,300.00
6 6% $ 6,300.00 $ -
Totals 100% $ 105,000.00

Year 1 Year 2 Year 3 Year 4


Increase in EBIT $ 35,000.00 $ 35,000.00 $ 35,000.00 $ 35,000.00
Taxes at 34% $ 11,900.00 $ 11,900.00 $ 11,900.00 $ 11,900.00
NOPAT $ 23,100.00 $ 23,100.00 $ 23,100.00 $ 23,100.00
Depreciation $ 21,000.00 $ 33,600.00 $ 19,950.00 $ 12,600.00
Net Operating CF $ 44,100.00 $ 56,700.00 $ 43,050.00 $ 35,700.00

Part c:
Decrease in NOWC $ 5,000.00

Cash Inflow from the Sale $ 10,000.00


Capital Gain:
Selling Price $ 10,000.00
Less: BV $ (6,300.00)
Capital Gain $ 3,700.00
Tax on Cap. Gain $ 1,258.00 $ (1,258.00)
Net Cash Flow from Sale $ 8,742.00

Total Nonoperating CF in Year 5 = $ 13,742.00

Part d:
Year Project FCFs
0 $ (110,000.00)
1 $ 44,100.00
2 $ 56,700.00
3 $ 43,050.00
4 $ 35,700.00
5 $ 48,392.00
WACC = 15.00%
NPV $ 43,998.21
PI 1.4000
IRR 31.2545%
MIRR 23.0049%
Example 2: The R.T. Kleinman Corp. is considering selling one of its old assembly machines. The
machine, purchased for $40,000 five years ago, had an expected life of 10 years and is
being depreciated by MACRS as 10 year property. Assume Kleinman could sell this old
machine for $45,000. Also, assume a 34% marginal tax rate. How would this sale
affect the initial outlay for a new machine to replace this one?

Year MACRS % Depreciation Book Value


0 $ 40,000.00
1 10% $ 4,000.00 $ 36,000.00
2 18% $ 7,200.00 $ 28,800.00
3 14% $ 5,600.00 $ 23,200.00
4 12% $ 4,800.00 $ 18,400.00
5 9% $ 3,600.00 $ 14,800.00
6 7% $ 2,800.00 $ 12,000.00
7 7% $ 2,800.00 $ 9,200.00
8 7% $ 2,800.00 $ 6,400.00
9 7% $ 2,800.00 $ 3,600.00
10 6% $ 2,400.00 $ 1,200.00
11 3% $ 1,200.00 $ -
Totals 100% $ 40,000.00

Cash Inflow from the Sale $ 45,000.00


Capital Gain:
Selling Price $ 45,000.00
Less: BV $ (14,800.00)
Capital Gain $ 30,200.00
Tax on Cap. Gain $ 10,268.00 $ (10,268.00)
Net Cash Flow from Sale $ 34,732.00

Problem 11-15: The Bartram Pulley Company must decide between two mutually exclusive investment
projects. Each project costs $6,750 and has an expected life of 3 years. Annual net
cash flows begin 1 year after the initial investment is made and have the following
probability distributions:

Project A Project B
Probability Net Cash Flows Probability Net Cash Flows
20% $ 6,000.00 20% $0.00
60% $ 6,750.00 60% $ 6,750.00
20% $ 7,500.00 20% $ 18,000.00

BPC has decided to evaluate the riskier project at a 12% rate and the less risky
project at a 10% rate.
a. What is the expected value of the annual net cash flows from each project? What
is the coefficient of variation?
b. What is the risk-adjusted NPV of each project?
c. If it were known that Project B was negatively correlated with other cash flows of the
firm, whereas Project A was positively correlated, how would this knowledge affect the
decision? If Project B's cash flows were negatively correlated with gross domestic
product (GDP), would that influence your assessment of its risk?

Part a: Project A
Probability Net Cash Flows Prob.x Net CF Prob.x((CF - Exp. CF)^2)
20% $ 6,000.00 $ 1,200.00 $ 112,500.00
60% $ 6,750.00 $ 4,050.00 $ -
20% $ 7,500.00 $ 1,500.00 $ 112,500.00
Expected Net CF $ 6,750.00 $ 225,000.00 Variance
Std. Deviation $ 474.34
CV 0.0703

Project B
Probability Net Cash Flows Prob.x Net CF Prob.x((CF - Exp. CF)^2)
20% $0.00 $0.00 $ 11,704,500.00
60% $ 6,750.00 $ 4,050.00 $ 486,000.00
20% $ 18,000.00 $ 3,600.00 $ 21,424,500.00
Expected Net CF $ 7,650.00 $ 33,615,000.00 Variance
Std. Deviation $ 5,797.84
CV 0.7579

Part b:
Year Project A Project B
0 $ (6,750.00) $ (6,750.00)
1 $ 6,750.00 $ 7,650.00
2 $ 6,750.00 $ 7,650.00
3 $ 6,750.00 $ 7,650.00
WACC 10.00% 12.00%
Risk-Adj. NPV = $ 10,036.25 $ 11,624.01
NPV Profile

cal analysis
se in EBIT of

expected life of
MACRS as 5
Year 5
$ 35,000.00
$ 11,900.00
$ 23,100.00
$ 11,550.00
$ 34,650.00
achines. The
10 years and is
ould sell this old

ve investment
Annual net

oject? What
sh flows of the
dge affect the
FIN 5080-Jack De Jong: Chapter 11: Cash Flow Estimation and Risk Analysis
Quiz #16: May 31, 2011

1. As a member of UA Corporation’s financial staff, you must estimate the Year 1 cash flow for a
proposed project with the following annual data: sales revenue of $42,500, depreciation
expense of $10,000, cash operating expenses of $17,000, interest expense of $4,000, and a
marginal tax rate of 35.00%. What is UA’s Year 1 cash flow?

Sales Revenue $ 42,500.00


Cash Operating Expenses $ 17,000.00
Depreciation $ 10,000.00
EBIT $ 15,500.00
Less: Taxes at 35% $ (5,425.00)
Add: Depreciation $ 10,000.00
Net Operating Cash Flow $ 20,075.00

2. Marshall–Miller & Company is considering the purchase of a new machine for $50,000 installed.
The machine will be depreciated by MACRS as 5 year property. The firm expects to operate
the machine for 4 years and then to sell it for $12,500. If the marginal tax rate is 40.00%, what
will the after–tax salvage value be when the machine is sold at the end of Year 4?

Year MACRS % Depreciation Book Value


0 $ 50,000.00
1 20% $ 10,000.00 $ 40,000.00
2 32% $ 16,000.00 $ 24,000.00
3 19% $ 9,500.00 $ 14,500.00
4 12% $ 6,000.00 $ 8,500.00
5 11% $ 5,500.00 $ 3,000.00
6 6% $ 3,000.00 $ -
Totals 100% $ 50,000.00

Cash Inflow from the Sale $ 12,500.00


Capital Gain:
Selling Price $ 12,500.00
Less: BV $ (8,500.00)
Capital Gain $ 4,000.00
Tax on Cap. Gain $ 1,600.00 $ (1,600.00)
Net Cash Flow from Sale $ 10,900.00

3. Your company, CSUS Inc., is considering a project whose data are shown below. Revenue and
cash operating expenses are expected to constant over the project’s 10 year expected
operating life; annual sales revenue is $42,500 and cash operating expenses are $25,000. The
new equipment’s cost and depreciable basis is $70,000 and it will be depreciated by MACRS
as 3 year property. The marginal tax rate is 35.00%. What is the project’s Year 4 cash flow?

Year MACRS % Depreciation Book Value


0 $ 70,000.00
1 33% $ 23,100.00 $ 46,900.00
2 45% $ 31,500.00 $ 15,400.00
3 15% $ 10,500.00 $ 4,900.00
4 7% $ 4,900.00 $ -
Totals 100% $ 70,000.00

Sales Revenue $ 42,500.00


Cash Operating Expenses $ 25,000.00
Depreciation $ 4,900.00
EBIT $ 12,600.00
Less: Taxes at 35% $ (4,410.00)
Add: Depreciation $ 4,900.00
Net Operating Cash Flow $ 13,090.00
FIN 5080 - Jack De Jong: Review for Final Exam
May 31, 2011

MB8:
Problem 11-7:
Part a:
Equipment Cost $ 70,000.00
Modification $ 15,000.00
Total Cost of Equip. $ 85,000.00
Increase NOWC $ 4,000.00
Initial Outlay $ 89,000.00
Part b:
Year MACRS % Depreciation Book Value
0 $ 85,000.00
1 33% $ 28,050.00 $ 56,950.00
2 45% $ 38,250.00 $ 18,700.00
3 15% $ 12,750.00 $ 5,950.00
4 7% $ 5,950.00 $ -
Totals 100% $ 85,000.00

Year 1 Year 2 Year 3


Reduced Labor $ 25,000.00 $ 25,000.00 $ 25,000.00
Increased Deprec. $ (28,050.00) $ (38,250.00) $ (12,750.00)
EBIT $ (3,050.00) $ (13,250.00) $ 12,250.00
Less: Taxes at 40% $ 1,220.00 $ 5,300.00 $ (4,900.00)
Add: Deprec. $ 28,050.00 $ 38,250.00 $ 12,750.00
Net Oper. CFs $ 26,220.00 $ 30,300.00 $ 20,100.00

Part c:
Decrease in NOWC $ 4,000.00

Cash Inflow from the Sale $ 30,000.00


Capital Gain:
Selling Price $ 30,000.00
Less: BV $ (5,950.00)
Capital Gain $ 24,050.00
Tax on Cap. Gain $ (9,620.00) $ (9,620.00)
Net Cash Flow from Sale $ 20,380.00

Total Nonoperating CF in Year 3 = $ 24,380.00


Part d:
Year Cash Flows
0 $ (89,000.00)
1 $ 26,220.00
2 $ 30,300.00
3 $ 44,480.00
WACC = 10.00%
NPV ($6,703.83) Reject the project

Problem 11-10:
Initial Outlay $ 82,500.00

Year MACRS % Depreciation Book Value


0 $ 82,500.00
1 20% $ 16,500.00 $ 66,000.00
2 32% $ 26,400.00 $ 39,600.00
3 19% $ 15,675.00 $ 23,925.00
4 12% $ 9,900.00 $ 14,025.00
5 11% $ 9,075.00 $ 4,950.00
6 6% $ 4,950.00 $ -
Totals 100% $ 82,500.00

Year 1 Year 2 Year 3 Year 4


EBD $ 27,000.00 $ 27,000.00 $ 27,000.00 $ 27,000.00
Increased Deprec. $ (16,500.00) $ (26,400.00) $ (15,675.00) $ (9,900.00)
EBIT $ 10,500.00 $ 600.00 $ 11,325.00 $ 17,100.00
Less: Taxes at 40% $ (4,200.00) $ (240.00) $ (4,530.00) $ (6,840.00)
Add: Deprec. $ 16,500.00 $ 26,400.00 $ 15,675.00 $ 9,900.00
Net Oper. CFs $ 22,800.00 $ 26,760.00 $ 22,470.00 $ 20,160.00

Year 5 Year 6 Year 7 Year 8


EBD $ 27,000.00 $ 27,000.00 $ 27,000.00 $ 27,000.00
Increased Deprec. $ (9,075.00) $ (4,950.00) $ - $ -
EBIT $ 17,925.00 $ 22,050.00 $ 27,000.00 $ 27,000.00
Less: Taxes at 40% $ (7,170.00) $ (8,820.00) $ (10,800.00) $ (10,800.00)
Add: Deprec. $ 9,075.00 $ 4,950.00 $ - $ -
Net Oper. CFs $ 19,830.00 $ 18,180.00 $ 16,200.00 $ 16,200.00

WACC = 12.00%
NPV = $22,329.39 Accept Project

Problem 7-19:
Year Growth Rate Dividend Cash Flows
0 $1.6000
1 20.00% $1.9200 $1.9200
2 20.00% $2.3040 $63.3600
3 6.00% $2.4422

Required Rate of Return on TTC = 10.00%


Price at End of Year 2 = D3/(rs - g) = $61.0560

Find Price Today using NPV: I 10.00%


NPV $54.11 Part a
Div. Yield = D1/P0 hat 3.55%
CGY = rs - Div. Yield 6.45%

Part b: Price increase, div. yield decrease, CGY increase


Part c: CGY = g = 6% and div. yield = 10% - 6% = 4%
Part d: Investor preferences for div. or CGs

You might also like